2
$\begingroup$

Let be given natural numbers N_1,N_2,N_3,...,N_k such that for every prime p less or equal k set N_1,N_2,N_3,...,N_k does not contain all reminders modulo p. Is it right that there exists number X such that all X+N_1,X+N_2,X+N_3,...,X+N_k are prime? I think it must follow from some theorems about prime numbers in arithmetical progression.

$\endgroup$
4
  • 8
    $\begingroup$ If you could solve the case N_1=1, N_2=3 I'd be very interested ... $\endgroup$
    – gowers
    Mar 9, 2010 at 11:21
  • $\begingroup$ I don't think the OP is asking for infinitely many. I can settle your case by exhibiting X=2, can't I? $\endgroup$
    – TonyK
    Mar 9, 2010 at 11:25
  • $\begingroup$ Even this variant is not going to work easily: simply look for pairs of consecutive primes arbitrarily far apart from each other to find different twin primes. $\endgroup$
    – damiano
    Mar 9, 2010 at 11:29
  • 8
    $\begingroup$ Yes---perhaps one should instead do $N_1=1$, $N_2=3$, $N_3=N+1$ and $N_4=N+3$ for some super-large $N$ congruent to 0 mod 6. This definitely forces infinitely many twin primes. $\endgroup$ Mar 9, 2010 at 11:36

1 Answer 1

6
$\begingroup$

(This question has been killed in the comments, but it is still lacking the useful pointers.)

This is a weak form of the Hardy-Littlewood Conjecture which moreover predicts an asymptotic density for the number of such prime k-tuplets. Special cases of this include: twin primes, cousin primes, sexy primes, prime quadruplets, quintuplets, and sextuplets. While your conjecture is much weaker than Hardy-Littlewood, Kevin Buzzard's trick in the comments shows that it globally implies the infinitude of prime k-tuplets for any admissible pattern.

As far as I know, the infinitude of prime k-tuplets is an open problem for all fixed admissible patterns with k ≥ 2. Note that the Green-Tao Theorem falls short of proving any instance of this since the step size of the arithmetic progressions is not fixed. (Even the Tao-Ziegler Theorem falls short since the polynomials are required to have vanishing constant term.)

$\endgroup$
1
  • 4
    $\begingroup$ It has been conjectured that, for any $k>0$ and $n>2$, there are at least as many primes between 2 and $n$ (inclusive) as there are between $k+2$ and $k+n$. About 35 years ago, Hensley and Richards proved that this conjecture is incompatible with the prime $k$-tuples conjecture. It's the only example that comes to my mind of two plausible conjectures being proved incompatible. It's generally believed now that prime $k$-tuples is true and the other conjecture is false. $\endgroup$ Mar 9, 2010 at 23:24

Your Answer

By clicking “Post Your Answer”, you agree to our terms of service and acknowledge you have read our privacy policy.

Not the answer you're looking for? Browse other questions tagged or ask your own question.